10
$\begingroup$

Being far from analysis, I recently learned about the Invariant subspace problem and came up with the following (perhaps simple or well-known) question.

Let $H$ be a separable complex Hilbert space and $T:H\to H$ a bounded operator. Assume that the spectrum of $H$ is $\{0\}$, i.e. $T-\lambda I$ has a bounded inverse for every $\lambda\in\mathbb C\setminus\{0\}$. In finite dimensions, this would imply that $T$ is nilpotent ($T^n=0$ for some $n$). I wonder if there is something similar in the infinite dimensional case. The precise formulation I have in mind is the following.

It is easy to see that there is a maximal subspace $X\subset H$ such that $T(X)=X$. This is a purely set-theoretic fact, a sort of explicit construction is the intersection of the images of iterations of $T$ up to and beyond infinity (via transfinite induction).

Question: can it happen that $X\ne\{0\}$?

Here are some observations that I made:

  • $T$ cannot be onto. (I derived this from some random Wikipedia quotes so there are high chances of error; please correct me if I am wrong.) It follows that a nontrivial $X$ cannot be closed.

  • If there is an example, then there is one where $X$ is dense. Just take the closure of $X$ for $H$.

  • It is possible that $T(H)$ is dense. My example is the shift in $\ell^2(\mathbb Z)$ composed with a mutiplication by a positive function (sequence) that goes to zero at both ends. Again, please correct me if I am wrong.

$\endgroup$
2
  • 2
    $\begingroup$ Bounded operators with spectrum $\\{0\\}$ are called quasinilpotent. I don't know much about them, but I guess that knowing the terminology will help you to find some references. $\endgroup$
    – Ian Morris
    May 15, 2010 at 9:23
  • 1
    $\begingroup$ Incidentally, Apostol and Voiculescu proved that every quasinilpotent operator is a norm limit of nilpotent operators. See for example jstor.org/pss/2042882 $\endgroup$ May 17, 2010 at 6:53

2 Answers 2

6
$\begingroup$

Perhaps I am missing something. Isn't the example you mention quasinilpotent and and maps the finitely non zero sequences onto themselves?

I am not an expert on invariant subspaces, but I think it is widely believed among experts that the answer to the question is the same for quasinilpotent operators as for general operators.

$\endgroup$
4
  • $\begingroup$ Sergei is asking for a large invariant subspace which may not be closed. I think that the existence of a large, dense $X$ such that $TX=X$ is not constrained by the ordinary invariant subspace problem, which asks for closed, proper invariant subspaces. $\endgroup$
    – Ian Morris
    May 15, 2010 at 13:49
  • $\begingroup$ That is what I thought, Ian. The finitely non zero sequences $X$ satisfy $TX=X$ for the operator Sergei mentioned (indeed, for any right shift with all weights non zero, but you need the null condition at $\pm \infty$ to make it quasinilpotent). Am I misinterpreting Sergei's question? $\endgroup$ May 15, 2010 at 14:17
  • $\begingroup$ Sorry, ignore my comment. $\endgroup$
    – Ian Morris
    May 15, 2010 at 16:37
  • $\begingroup$ Yes, this is exactly what I asked. It is simple as I suspected, I just did not see it. Thanks! $\endgroup$ May 15, 2010 at 17:23
2
$\begingroup$

Just some remarks. A natural example of a quasinilpotent operator, which may be of interest to you, is Tu(x):=∫0xu(s)ds on C[0,1], or on Lp[0,1], for 1≤p≤∞. In all these cases T is compact. However note that a noncompact example can be easily defined e.g. on a Hilbert sum of infinitely many Hilbert copies of a Hilbert space H, as the direct sum of infinitely many copies of a non-zero quasinilpotent T:H→H (that is, just taking {hj} to {Thj}).

$\endgroup$
2
  • 1
    $\begingroup$ Another noncompact example is $\left( \begin{matrix} 0 & I \\ 0 & 0 \end{matrix} \right)$ acting on $H\oplus H$ if $H$ is infinite dimensional. (I'm just taking the Kronecker product in the reverse order.) $\endgroup$ May 17, 2010 at 1:37
  • 1
    $\begingroup$ Actually this is the simplest example of a direct sum of infinitely many copies of a non-zero quasinilpotent $T$ (on a 2-dimensional space) :) $\endgroup$ Jul 5, 2017 at 19:48

Your Answer

By clicking “Post Your Answer”, you agree to our terms of service and acknowledge you have read our privacy policy.

Not the answer you're looking for? Browse other questions tagged or ask your own question.